LSAT and Law School Admissions Forum

Get expert LSAT preparation and law school admissions advice from PowerScore Test Preparation.

General questions relating to LSAT Logical Reasoning.
 AspiringLawyer
  • Posts: 11
  • Joined: Oct 07, 2017
|
#41737
From October 2012, Section 3, Question 10
"Which one of the following is most strongly supported by the information above?"
What question type is this? I would personally rephrase as "If the information above is considered to be true, which answer choice does it most support?"
This tells me at least it is Family 1, so I feel it is closest to a Must Be True, but not a full-fledged Must Be True. Thoughts?
 Adam Tyson
PowerScore Staff
  • PowerScore Staff
  • Posts: 5153
  • Joined: Apr 14, 2011
|
#41742
Right you are! We treat this type as being in the Must Be True category, but it is the weaker variant, where the correct answer doesn't absolutely have to be true, but it is the one most likely to be based on the support provided in the stimulus. You can still view it as you would a Must Be True question, and remind yourself that your job is to pick the BEST answer, even if it isn't perfect. Or, you can view it as a "most likely but not absolutely required". Typically, the four wrong answers will have no support from the stimulus, or only very tenuous support, while the correct answer will appear to be very well supported.

Nice work, good analysis!

Get the most out of your LSAT Prep Plus subscription.

Analyze and track your performance with our Testing and Analytics Package.